Don't be more, be less!

Algebra Level 4

Let x , y , z , t x, y, z, t be positive real numbers such that 1 1 + x + 1 1 + y + 1 1 + z + 1 1 + t 3 \dfrac { 1 }{ 1+x } +\dfrac { 1 }{ 1+y } +\dfrac { 1 }{ 1+z } +\dfrac { 1 }{ 1+t } \ge 3 .

If the maximum value of x y z t xyzt can be expressed as p q \frac pq , where p p and q q are coprime positive integers, then find p + q p + q .


The answer is 82.

This section requires Javascript.
You are seeing this because something didn't load right. We suggest you, (a) try refreshing the page, (b) enabling javascript if it is disabled on your browser and, finally, (c) loading the non-javascript version of this page . We're sorry about the hassle.

4 solutions

Relevant wiki: Jensen's Inequality

Define f ( x ) = 1 1 1 + x f(x)=1-\dfrac{1}{1+x} and notice that the given inequality is similar to c y c l i c ( 1 1 1 + x ) 1 \displaystyle \sum_{cyclic} \left(1-\dfrac{1}{1+x}\right)\le 1 . Now since given quantities are positive we have f ( x ) > 0 f''(x)\gt 0 which implies f f is convex throughout the domain. Thus if we use jensen's inequality,

f ( x ) + f ( y ) + f ( z ) + f ( t ) 4 f ( x + y + z + t 4 ) = 4 4 1 + x + y + z + t 4 \displaystyle f(x)+f(y)+f(z)+f(t)\ge 4f\left(\dfrac{x+y+z+t}{4}\right)=4-\dfrac{4}{1+\dfrac{x+y+z+t}{4}} , Now if α = x y z t \alpha = xyzt then we can deduce by AM-GM that x + y + z + t 4 α 4 \dfrac{x+y+z+t}{4}\ge \sqrt[4]{\alpha} and thus 1 1 + x + y + z + t 4 1 α 4 + 1 \displaystyle -\dfrac{1}{1+\dfrac{x+y+z+t}{4}} \ge -\dfrac{1}{\sqrt[4]{\alpha}+1} .

This hence reduces to 1 c y c l i c ( 1 1 1 + x ) 4 4 1 + α 4 \displaystyle 1\ge \sum_{cyclic} \left(1-\dfrac{1}{1+x}\right) \ge 4-\dfrac{4}{1+\sqrt[4]{\alpha}} , on solving we have α = x y z t 1 81 \alpha=xyzt\le \dfrac{1}{81} making the answer 82 \boxed{82} .

Nice solution! Never expect Jensen's inequality being used here!

Steven Jim - 4 years ago

Log in to reply

You must expect the unexpected ! ;)

Log in to reply

Yeah... I'm really impressed! :) Thanks for the lesson!

Steven Jim - 4 years ago

By the way, do you think you can solve this by only using Cauchy and/or AM - GM?

Steven Jim - 4 years ago

Log in to reply

@Steven Jim I would think about doing it by AM-GM, you must change the part x y \dfrac{x}{y} as a b \dfrac{a}{b} or any two variables you like. It's confusing that the dummy variables coincide with the original variables

Log in to reply

@Aditya Narayan Sharma Oh, really? I'm sorry if I made that mistake. I was a little bit sleepy when I posted the problem. Thanks for finding out!

Steven Jim - 4 years ago

f ( x ) = 2 ( x + 1 ) 3 f''(x)=-\dfrac{2}{(x+1)^3}

How this is greater than zero ?

Kushal Bose - 4 years ago

Log in to reply

@Kushal Bose , Hey thanks for pointing out, I will modify that as soon as possible.

@Aditya Narayan Sharma Hey, check this one out!

Steven Jim - 4 years ago

Applying the AM-GM inequality, we have 1 1 + x = ( 1 1 1 + y ) + ( 1 1 1 + z ) + ( 1 1 1 + t ) = y 1 + y + z 1 + z + t 1 + t 3 y z t ( 1 + y ) ( 1 + z ) ( 1 + t ) 3 \dfrac{1}{1+x}=\left(1-\dfrac{1}{1+y}\right)+\left(1-\dfrac{1}{1+z}\right)+\left(1-\dfrac{1}{1+t}\right)=\dfrac{y}{1+y}+\dfrac{z}{1+z}+\dfrac{t}{1+t}\ge3\sqrt[3]{\dfrac{yzt}{(1+y)(1+z)(1+t)}} Similarly, 1 1 + y 3 x z t ( 1 + x ) ( 1 + z ) ( 1 + t ) 3 ; 1 1 + z 3 x y t ( 1 + x ) ( 1 + y ) ( 1 + t ) 3 ; 1 1 + t 3 x y z ( 1 + x ) ( 1 + y ) ( 1 + z ) 3 \dfrac{1}{1+y}\ge3\sqrt[3]{\dfrac{xzt}{(1+x)(1+z)(1+t)}};\dfrac{1}{1+z}\ge3\sqrt[3]{\dfrac{xyt}{(1+x)(1+y)(1+t)}};\dfrac{1}{1+t}\ge3\sqrt[3]{\dfrac{xyz}{(1+x)(1+y)(1+z)}}

Multiple side by side all above inequalities, we get 1 ( 1 + x ) ( 1 + y ) ( 1 + z ) ( 1 + t ) 81 x y z t ( 1 + x ) ( 1 + y ) ( 1 + z ) ( 1 + t ) \dfrac{1}{(1+x)(1+y)(1+z)(1+t)}\ge\dfrac{81xyzt}{(1+x)(1+y)(1+z)(1+t)}

Or x y z t 1 81 xyzt\le\dfrac{1}{81} .

The equality holds iff x = y = z = t = 1 3 x=y=z=t=\dfrac{1}{3} .

Good solution!

Steven Jim - 4 years ago

A very clean solution!

Phan Nhật Huy - 12 months ago
Utsav Playz
Oct 2, 2019

We can maximize this equation as we suppose WLOG, that x=y=z=t.

We observe that,

4/1+x >= 3

This gives us

x >= 1/3

So the value of xyzt = x^4

So, if we take the original equation, and raise both sides to the power of 4, we get.

x^4 >= 1/81 where g.c.d(1, 81) = 1

Hence, gives us the answer 1+81 = 82

Su Ku
May 28, 2017

Using symmetry arguments we can say that at maximum x y z t , x = y = z = t xyzt, x = y = z = t . This means x < = 1 3 x <= \frac{1}{3} . Hence maximum x y z t = 1 81 xyzt = \frac{1}{81} and hence the answer is 82.

0 pending reports

×

Problem Loading...

Note Loading...

Set Loading...